Thứ Sáu, 1 tháng 3, 2019

Bất đẳng thức tuyển sinh lớp 10 chọn lọc


Trong bài viết này, tác giả giới thiệu một số bài BĐT nhẹ nhàng nhưng ý tưởng tương đối mới, mức độ phù hợp với đề thi tuyển sinh vào lớp 10 Chuyên.

Bài 1. Cho các số thực dương $a,b,c$ thỏa mãn
${{a}^{2}}+{{b}^{2}}+{{c}^{2}}=3$ và $\min \{a+b,b+c,c+a\}>\sqrt{2}$.
a) Chứng minh rằng $a,b,c$ là độ dài ba cạnh của một tam giác.
b) Biết rằng diện tích $S$ của tam giác nói trên có thể tính theo công thức
$16{{S}^{2}}=2({{a}^{2}}{{b}^{2}}+{{b}^{2}}{{c}^{2}}+{{c}^{2}}{{a}^{2}})-({{a}^{4}}+{{b}^{4}}+{{c}^{4}}).$
Tìm giá trị lớn nhất của $S.$

Lời giải.

a) Đặt $c$ là số lớn nhất trong các số $a,b,c$. Để chứng minh $a,b,c$ là các cạnh của tam giác, ta chỉ cần làm rõ $c<a+b.$
Theo đề bài thì $a+b>\sqrt{2}$ nên ${{a}^{2}}+{{b}^{2}}\ge \frac{1}{2}{{(a+b)}^{2}}>1$, suy ra ${{c}^{2}}<2$ hay $c<\sqrt{2}$.
Do đó, $a+b>c$ hay $a,b,c$ là ba cạnh của tam giác.
b) Theo công thức đã cho thì
$16{{S}^{2}}={{({{a}^{2}}+{{b}^{2}}+{{c}^{2}})}^{2}}-2({{a}^{4}}+{{b}^{4}}+{{c}^{4}})\le \frac{1}{3}{{({{a}^{2}}+{{b}^{2}}+{{c}^{2}})}^{2}}=3.$
Vậy giá trị lớn nhất của $S$ là $\frac{\sqrt{3}}{4}$, đạt được khi $a=b=c=1.$

Bài 2. Cho $a,b,c$ là các số thực dương có tổng bằng $1.$ Chứng minh rằng
\[\frac{a}{b}+\frac{b}{c}+\frac{c}{a}+\frac{b}{a}+\frac{c}{b}+\frac{a}{c}+6\ge \sqrt{\frac{8}{a}-8}+\sqrt{\frac{8}{b}-8}+\sqrt{\frac{8}{c}-8}.\]

Lời giải.

Ghép cặp và dùng BĐT Cô-si cho hai số:
$\frac{a}{b}+\frac{c}{b}+2=\frac{a+c}{b}+2\ge 2\sqrt{\frac{2(a+c)}{b}}=2\sqrt{\frac{2(1-b)}{b}}=\sqrt{\frac{8}{b}-8}$.

Bài 3. Cho $x,y,z$ là các số thực dương thỏa mãn điều kiện $x+y+z=\frac{1}{x}+\frac{1}{y}+\frac{1}{z}$. Chứng minh rằng \[\sqrt{2}(x+y+z)\ge \sqrt{xy+1}+\sqrt{yz+1}+\sqrt{zx+1}.\]

Lời giải.

Theo BĐT Cô-si thì \[(x+\frac{1}{y})+2y\ge 2\sqrt{2y\left( x+\frac{1}{y} \right)}=2\sqrt{2}\cdot \sqrt{xy+1},\] suy ra
$x+2y+\frac{1}{y}\ge 2\sqrt{2}\cdot \sqrt{xy+1}$.
Thực hiện tương tự và cộng lại, ta có
$3x+3y+3z+\frac{1}{x}+\frac{1}{y}+\frac{1}{z}\ge 2\sqrt{2}\left( \sqrt{xy+1}+\sqrt{yz+1}+\sqrt{zx+1} \right)$.
Chú ý rằng $\frac{1}{x}+\frac{1}{y}+\frac{1}{z}=x+y+z$ nên vế trái chính là $4(x+y+z).$

Bài 4. Cho $a,b,c$ là các số thực dương thỏa mãn $abc=1.$ Chứng minh rằng
${{\left( a+\frac{1}{b} \right)}^{2}}+{{\left( b+\frac{1}{c} \right)}^{2}}+{{\left( c+\frac{1}{a} \right)}^{2}}\ge 3(a+b+c+1).$

Lời giải.

Theo BĐT quen thuộc ${{x}^{2}}+{{y}^{2}}+{{z}^{2}}\ge xy+yz+zx$, ta có
VT \ge ( a+\frac{1}{b}) ( b+\frac{1}{c}) + ( b+\frac{1}{c}) ( c+\frac{1}{a}) + ( c+\frac{1}{a} ) ( a+\frac{1}{b}) $
\[=ab+bc+ca+\frac{a}{c}+\frac{c}{b}+\frac{b}{a}+3+a+b+c \]
Ta lại có $ab+\frac{b}{a}\ge 2b$ nên áp dụng tương tự và cộng lại thì
$ab+bc+ca+\frac{b}{a}+\frac{a}{c}+\frac{c}{b}\ge 2(a+b+c).$
Thay vào, ta có đpcm.

Bài 5. Cho $a,b,c$ là các số thực dương thỏa mãn ${{x}^{2}}+{{y}^{2}}+{{z}^{2}}=2(xy+yz+zx).$
Tìm giá trị nhỏ nhất của biểu thức $P=x+y+z+\frac{1}{2xyz}$.

Lời giải. 

Đẳng thức đã cho viết lại thành ${{x}^{2}}+{{y}^{2}}+{{z}^{2}}-2xy-2yz+2yz=4yz$ hay
${{(y+z-x)}^{2}}=4yz$.
Không mất tính tổng quát, có thể giả sử $x$ là số nhỏ nhất, khi đó $y+z-x>0.$ Ta có
$P=(y+z-x)+2x+\frac{2}{x{{(y+z-x)}^{2}}}$.
Chú ý rằng $2x+\frac{2}{x{{(y+z-x)}^{2}}}\ge \frac{4}{y+z-x}$ nên
$P\ge y+z-x+\frac{4}{y+z-x}\ge 4$.
Giá trị nhỏ nhất là $4$, đẳng thức xảy ra khi $y=2,x=z=\frac{1}{2}.$

Bài 6. Cho $a,b,c$ là các số thực dương thỏa mãn $a\ge 1\ge b.$ Tìm giá trị nhỏ nhất của
\[P=(a+b+c-1)\left( \frac{1}{a}+\frac{1}{b}+\frac{1}{c}-1 \right).\]

Lời giải.

Theo đề bài thì $a\ge 1\ge b$ nên $(a-1)(b-1)\le 0$ hay $a+b\ge 1+ab\Leftrightarrow \frac{1}{a}+\frac{1}{b}\ge 1+\frac{1}{ab}$.
Do đó $a+b+c-1\ge 1+ab+c-1=ab+c$ và $\frac{1}{a}+\frac{1}{b}+\frac{1}{c}-1\ge \frac{1}{ab}+\frac{1}{c}$. Suy ra
$P\ge (ab+c)\left( \frac{1}{ab}+\frac{1}{c} \right)=2+\frac{ab}{c}+\frac{c}{ab}\ge 2+2\sqrt{\frac{abc}{abc}}=4.$
Do đó, giá trị nhỏ nhất cần tìm là $4$, đạt được khi $a=b=c=1.$ 

Hệ thặng dư đầy đủ

Hệ thặng dư đầy đủ là một trong các cơ sở quan trọng của số học. Nhờ nó, ta có thể chứng minh được dễ dàng định lý Fermat nhỏ, định lý Wilson.
Nói sơ qua về định nghĩa, với $m>1$ là số nguyên dương. Tập hợp các số nguyên $\{a_1, a_2, \ldots, a_m \}$ là hệ thặng dư đầy đủ $\pmod m$ nếu như các số đó đôi một không đồng dư với nhau theo $\pmod m$.
Trong bài viết nhỏ này, chúng ta sẽ cùng tìm hiểu một số ví dụ vận dụng hệ thặng dư đầy đủ trong việc giải các bài toán số học, tổ hợp.


Bài 1. Cho số nguyên dương $n\ge 3$ và đa giác đều $n$ đỉnh. Trên mỗi đỉnh của đa giác, người ta chọn điền các số nguyên dương nào đó. Tiếp theo, với mỗi đường chéo hoặc cạnh bất kỳ, người ta viết số dư của tổng hai số được điền trên hai đầu mút của đoạn thẳng đó khi chia cho $m=\frac{n(n-1)}{2}.$ Chứng minh rằng với mọi $n$ không là số chính phương và chia $4$ dư $0,1$ thì không tồn tại cách điền như thế sao cho tất các các số được điền trên đường chéo và cạnh là đôi một phân biệt.

Lời giải.

Theo giả thiết thì $m$ số điền trên cạnh, đường chéo phải lập được thành hệ thặng dư đầy đủ $\bmod m.$
Giả sử có cách điền các số trên đa giác, trong đó có $a$ số chẵn, $b$ số lẻ với $a+b=n$. Khi đó, việc tính tổng các số trong $n$ số đó sẽ sinh ra $ab$ số lẻ và $\frac{{{a}^{2}}+{{b}^{2}}-a-b}{2}$ số chẵn.
Vì $n\equiv 0,1\text{ }(\bmod 4)$ nên $m=\frac{n(n-1)}{2}$ là số chẵn, để có được hệ thặng dư thì
$ab=\frac{{{a}^{2}}+{{b}^{2}}-a-b}{2}\Leftrightarrow {{(a-b)}^{2}}=a+b=n$.
Tuy nhiên, $n$ không phải là số chính phương nên điều kiện trên không thỏa.



Bài 2.
a) Cho số nguyên tố $p$ và hai hệ thặng dư đầy đủ $\bmod p$ là $({{a}_{1}},{{a}_{2}},\ldots ,{{a}_{p}})$ và $({{b}_{1}},{{b}_{2}},\ldots ,{{b}_{p}})$.
Chứng minh rằng ${{a}_{1}}{{b}_{1}},{{a}_{2}}{{b}_{2}},\ldots ,{{a}_{p}}{{b}_{p}}$ không là hệ thặng dư đầy đủ $\bmod p.$
b) Tìm tất cả $n$ nguyên dương để tồn tại hai hệ thặng dư đầy đủ $\bmod n$ sao cho tổng tương ứng của chúng theo chỉ số cũng là hệ thặng dư đầy đủ mod $n.$

Lời giải.

a) Giả sử $p|{{a}_{r}},p|{{b}_{s}}$ với $r,s\in \{1,2,3\ldots p\}$ thì có hai trường hợp:
- Nếu $r\ne s$ thì ${{a}_{r}}{{b}_{r}},{{a}_{s}}{{b}_{s}}$ cùng chia hết cho $p$ nên không thỏa.
- Nếu $r=s$ thì có thể giả sử $r=s=p$, ta xét các số còn lại của hệ.
Ta cũng có 
$({{a}_{1}}{{b}_{1}})({{a}_{2}}{{b}_{2}})\ldots ({{a}_{p-1}}{{b}_{p-1}})=({{a}_{1}}{{a}_{2}}\ldots {{a}_{p-1}})({{b}_{1}}{{b}_{2}}\ldots {{b}_{p-1}})\equiv {{(-1)}^{2}}=1(\bmod p)$ 
theo định lý Wilson, điều này chứng tỏ các số ${{a}_{1}}{{b}_{1}},{{a}_{2}}{{b}_{2}},\ldots ,{{a}_{p}}{{b}_{p}}$ không lập được thành hệ thặng dư đầy đủ mod $p.$ (đpcm)
b) Số $n$ lẻ. Chỉ cần xét tổng các số tương tự như việc xét tích ở trên.

Bài 3. Gọi $P$ là tích các số nguyên tố không vượt quá $2018.$ Số nguyên dương $n$ được gọi là tốt nếu như nó chỉ có ước nguyên tố thuộc $P$ và không tồn tại $a,d\in {{\mathbb{Z}}^{+}}$ để $a,a+d,\ldots ,a+2018d$ đều nhỏ hơn $n$ và nguyên tố cùng nhau với $n.$
a) Chứng minh rằng $2018P$ là số tốt.
b) Chứng minh rằng $2018P$ là số tốt lớn nhất.

Lời giải.

a) Giả sử có $a,d\in {{\mathbb{Z}}^{+}}$ để $a,a+d,\ldots ,a+2018d$ đều nhỏ hơn $2018P$ và nguyên tố cùng nhau với $2018P.$ Vì $a+2018d<2018P$ nên $d<P$, chứng tỏ tồn tại số nguyên tố $p<2018$ để $(d,p)=1.$
Khi đó, các số $a,a+d,\ldots ,(p-1)d$ lập thành hệ thặng dư đầy đủ $\bmod p$ nên sẽ có số chia hết cho $p$; trong khi $p|2018P$, mâu thuẫn.
b) Giả sử có số tốt $m>2018P$, chọn $a=1,d=P$ thì dễ thấy $a,d$ thỏa mãn điều kiện đề cho, mâu thuẫn.

Bài 4.
a) Cho đa thức $P(x)$ hệ số nguyên và hai số nguyên $a,b>1$ nguyên tố cùng nhau. Giả sử khi $x\in \mathbb{Z},$ $P(x)$ chạy qua một hệ thặng dư đầy đủ mod $a,b$. Chứng minh rằng $P(x)$ cũng chạy qua một hệ thặng dư đầy đủ mod $ab.$
b) Cho đa thức $P(x)$ hệ số nguyên sao cho tồn tại $c\in \mathbb{Z},c>1$ để $P({{x}_{1}}),P({{x}_{2}}),\ldots ,P({{x}_{c}})$ không chia hết cho $c$ với ${{x}_{1}},{{x}_{2}},\ldots ,{{x}_{c}}$ là một hệ thặng dư đầy đủ $\bmod \text{ }c.$ Chứng minh rằng $P(x)$ không có nghiệm nguyên.

Lời giải.

a) Giả sử có $P({{x}_{1}}) \equiv {{a}_{1}} \pmod a,P({{x}_{2}})\equiv {{b}_{1}} \pmod b$, chọn ${{x}_{0}}$ thỏa mãn ${{x}_{0}} \equiv {{x}_{1}} \pmod a$ và ${{x}_{0}}\equiv {{x}_{2}} \pmod b$ (thặng dư Trung Hoa). Khi đó $P({{x}_{0}})-P({{x}_{1}})$ chia hết cho $a$ hay $P({{x}_{0}})\equiv {{a}_{1}}(\bmod a),P({{x}_{0}})\equiv {{b}_{1}}(\bmod b)$ nên
$P({{x}_{0}})\equiv c(\bmod ab)$
với $c$ xác định theo ${{a}_{1}},{{b}_{1}}$. Cho ${{a}_{1}},{{b}_{1}}$ lần lượt chạy qua các số $\{0,1,\ldots ,a-1\},\{0,1,\ldots ,b-1\}$ thì dễ thấy $c$ sẽ chạy qua $\{0,1,\ldots ,ab-1\}$.

b) Giả sử $P(x)$ có nghiệm nguyên $x=a$ thì đặt $P(x)=(x-a)Q(x).$
Khi $x$ chạy qua một hệ thặng dư đầy đủ $\bmod \text{ }c$ thì $x-c$ cũng thế, tức là phải có một số chia hết cho $c$ hay $P(x)$ tương ứng cũng chia hết cho $c,$ mâu thuẫn.


Bài 5.
a) Cho số nguyên tố $p$ lẻ và với $k\in \{0,1,\ldots ,p-2\}$, đặt ${{S}_{k}}={{1}^{k}}+{{2}^{k}}+\cdots +{{(p-1)}^{k}}$. Chứng minh rằng ${{S}_{k}}$ chia hết cho $p.$
b) Cho đa thức $P(x)$ nguyên thỏa mãn $P(0)=0,P(1)=1$ và tồn tại số nguyên tố $p$ sao cho $P(n)$ chia $p$ dư $0$ hoặc $1$ với mọi $n\in \mathbb{Z}.$ Chứng minh rằng $\deg P\ge p-1.$

Lời giải.

a) Do $k<p-1$ nên tồn tại $a$ nguyên để ${{a}^{k}}\ne 1 \pmod p.$ Xét
${{a}^{k}}{{S}_{k}}={{a}^{k}}+{{(2a)}^{k}}+\cdots +{{((p-1)a)}^{k}}$.
Do $a,2a,\ldots ,(p-1)a$ lập thành hệ thặng dư thu gọn $\bmod p$ nên
${{a}^{k}}{{S}_{k}}\equiv {{S}_{k}}(\bmod p)$, suy ra ${{S}_{k}}\equiv 0(\bmod p).$
b) Giả sử $\deg P\le p-2$ thì ta viết $P(x)=\sum\limits_{i=0}^{n}{{{a}_{i}}{{x}^{i}}}$, trong đó $n\le p-2.$ Theo câu a thì
$P(1)+P(2)+\cdots +P(p-1)\equiv 0(\bmod p).$
Tuy nhiên, do $P(1)=1$ và $P(i)\equiv 0,1(\bmod p)$ nên vế trái không chia hết cho $p,$ mâu thuẫn.

Bài 6. Tìm tất cả các số nguyên dương $n$ sao cho tồn tại hoán vị $\{{{a}_{1}},{{a}_{2}},\ldots ,{{a}_{n}}\}$ của $\{1,2,\ldots ,n\}$ sao cho $\{{{a}_{1}}+1,{{a}_{2}}+2,\ldots ,{{a}_{n}}+n\}$ và $\{{{a}_{1}}-1,{{a}_{2}}-2,\ldots ,{{a}_{n}}-n\}$ cũng là hệ thặng dư đầy đủ mod $n.$

Lời giải.

Trước hết, xét hiệu \[0\equiv \sum\limits_{i=1}^{n}{{{a}_{i}}}-\sum\limits_{i=1}^{n}{i}\equiv \sum\limits_{i=1}^{n}{({{a}_{i}}-i)}\equiv \sum\limits_{i=1}^{n}{i}=\frac{n(n+1)}{2}(\bmod n)\] nên $n$ lẻ. Tiếp theo, ta cũng có
$\frac{4n(n+1)(2n+1)}{6}=2\left( \sum\limits_{i=1}^{n}{(a_{i}^{2}+{{i}^{2}})} \right)=\sum\limits_{i=1}^{n}{\left[ {{({{a}_{i}}+i)}^{2}}+{{({{a}_{i}}-i)}^{2}} \right]}\equiv \frac{2n(n+1)(2n+1)}{6}(\bmod n)$
nên $n$ không chia hết cho $3.$
Do đó $(n,6)=1.$ Ta chỉ cần xét ${{a}_{i}}\equiv 2i(\bmod n)$ là thỏa mãn đề bài.

Bài 7.
Có $m>1$ học sinh đứng trên vòng tròn. Cô giáo phát kẹo cho một học sinh nào đó, coi là học sinh thứ $1$ và theo chiều kim đồng hồ, cô lần lượt phát kẹo cho học sinh thứ ${{1}^{2}}+{{2}^{2}},{{1}^{2}}+{{2}^{2}}+{{3}^{2}},\ldots $ cứ như thế (một học sinh có thể nhận được kẹo nhiều lần, và khi đếm hết vòng thì cô tiếp tục đếm sang vòng mới). Số $m$ nguyên dương được gọi là tốt nếu học sinh nào cũng có kẹo sau sau hữu hạn lần phát kẹo.
a) Chứng minh rằng $m$ tốt chỉ có ước nguyên tố là $2$ hoặc $3.$
b) Tìm tất cả các số tốt.

Lời giải.

Thứ tự của học sinh thứ $n$ được phát kẹo là $f(n)=\frac{n(n+1)(2n+1)}{6}$.
Xét số $m$ tốt thì $f(n)$ có thể chạy qua hệ thặng dư đầy đủ $\bmod m.$ Ta thấy nếu có số nguyên tố $p|m$ thì $p$ cũng tốt.
Giả sử $m$ tốt có ước nguyên tố $p$ khác $2,3.$ Khi đó, $p\equiv 1,5(\bmod 6)$ và
$f(p+n)=\frac{(p+n)(p+n+1)(2p+2n+1)}{6}$
 $=\frac{n(n+1)(2n+1)}{6}+p\left[ \frac{(p+1)(2p+1)}{6}+{{n}^{2}}+pn+n \right]$
 $\equiv \frac{n(n+1)(2n+1)}{6}=f(n)\text{ }(\bmod p). $
Do đó, chỉ cần xét các số $n\in \{1,2,\ldots ,p\}$. Tuy nhiên,
$f(p)=f(p-1)+{{p}^{2}}\equiv f(p-1)\text{ }(\bmod p)$
nên các số $f(1),f(2),\ldots ,f(p)$ không thể lập thành hệ thặng dư đầy đủ mod $p.$
Tiếp theo, ta chứng minh rằng mọi số $m={{2}^{a}}{{3}^{b}}$ với $a,b\ge 0$ và không đồng thời bằng $0$ đều tốt.
Thật vậy,
Ta cần chỉ ra rằng với mọi $k\in \{0,1,\ldots ,m-1\}$ thì đều tồn tại $n$ để $m|f(n)-k$ hay
$\frac{n(n+1)(2n+1)-6k}{6}$ chia hết cho $m.$
Đặt $P(x)=2{{x}^{3}}+3{{x}^{2}}+x-6k$, cần chứng minh tồn tại $x$ để $P(x)$ chia hết cho ${{2}^{a+1}}\cdot {{3}^{b+1}}.$
Ta có $P(0)=-6k$ chia hết cho $2$ và $3$; ngoài ra ${P}'(x)=6{{x}^{2}}+6x+1$, không chia hết cho $2$ và $3$ nên theo bổ đề Hensel, ta luôn chọn được
$0\le {{x}_{1}}\le {{2}^{a+1}}-1$ để ${{2}^{a+1}}|P({{x}_{1}})$;
$0\le {{x}_{2}}\le {{3}^{b+1}}-1$ để ${{3}^{b+1}}|P({{x}_{2}})$.
Đến đây, vì $\gcd ({{2}^{a+1}},{{3}^{b+1}})=1$ nên theo thặng dư Trung Hoa, ta dễ dàng chọn được ${{x}_{0}}$ để $P({{x}_{0}})$ chia hết cho ${{2}^{a+1}}\cdot {{3}^{b+1}}.$

Vậy tất cả các số tốt cần tìm là $m={{2}^{a}}{{3}^{b}}$ với $a,b\ge 0$ và $a,b$ không đồng thời bằng $0$.

Thứ Bảy, 22 tháng 4, 2017

Dùng vị tự quay để giải một bài toán

Bài 1 (Trần Việt Hùng): Cho tam giác ABC nội tiếp (O) và đường tròn mixtilinear (D) ứng với góc A tiếp xúc với (O), AB, AC tại X, E, F. AX cắt đường tròn (D) tại K. L là điểm đối tâm của K đối với (D). EF cắt KL tại M. AL cắt (ABC) tại điểm thứ hai là N. Chứng minh XM vuông XN

Lời giải:


Ý tưởng là chứng minh $\angle MXA=\angle NXL$ Gọi MX cắt (D) tại R, Gọi A' là điểm đối tâm của A đối với (O) thì ta có X, L, A' thẳng hàng.
Ta có được: $\angle XAR=\angle XA'N$ ta cần chứng minh: $\Delta AXR \sim \Delta A'XN$ dùng phép vị tự quay tâm X ta đưa về bài toán $\Delta RXN \sim \Delta AXA'$
Mặt khác ta có: $\frac{RE}{RF}=\frac{XE}{XF}=\frac{KE}{KF}=\frac{LE}{LF}$ suy ra tứ giác RELF là tứ giác điều hòa suy ra L, R, A thẳng hàng.
Như vậy ta có: $\angle XNA= \angle XA'A$
Và $\angle XRN=\angle XKL=\angle XAA'$ ( do X là tâm vị tự của (D) biến thành (O) nên KL//AA' )
Như vậy ta có đpcm

Chủ Nhật, 26 tháng 3, 2017

Dùng thặng dư bậc hai để giải phương trình nghiệm nguyên

Các kiến thức cần nhớ:
$$-1 \equiv a^2 (mod p) \Leftrightarrow p \equiv 1 (mod 4)\\
2 \equiv a^2 (mod p) \Leftrightarrow p\equiv 1,7 (mod 8)\\
-2 \equiv a^2 (mod p) \Leftrightarrow p \equiv 1,3 (mod 8)$$

Ta sẽ xét các ví dụ dùng thặng dư bậc hai để chứng minh một số trường hợp vô nghiệm của phương trình Mordell:

Vd 1: Giải phương trình $y^2=x^3+7$ trên tập số tự nhiên.

Giải:

Giả sử tồn tại $(x,y)$ thỏa mãn.
Nếu $x$ chẵn thì $y^2 \equiv 7 (mod 8)$ ( Loại)
Nên $x$ lẻ và $y^2+1=(x+1)(x^2-2x+4)$

Vì $x$ lẻ nên $x^2-2x+4=(x-1)^2+3 \equiv 3 (mod 4)$ vì thế $x^2-2x+4$ phải có một ước nguyên tố $p \equiv 3 (mod 4)$ (nếu không thì $x^2-2x+4 \equiv 1 (mod 4)$)

Ta suy ra $p| y^2+1$ nên $-1 \equiv y^2 (mod p)$ không được do $p \equiv 3 (mod 4)$

Vd 2: Chứng minh phương trình $y^2=x^3-5$ vô nghiệm tự nhiên.

Giải:

Giả sử tồn tại, xét mod 4: $y^2\equiv x^3-1 (mod 4)$
Xét $y \equiv 0,1,2,3 (mod 4), x \equiv 0,1,2,3 (mod 4)$  nhận thấy chỉ có giá trị chung của $y^2 (mod 4)$ và $x^3-1 (mod 4)$ là $0$ vì thế $y$ chẵn và $x \equiv 1 (mod 4)$
vậy:
$y^2+4=x^3-1=(x-1)(x^2+x+1)$, $x^2+x+1 \ge 3 \equiv 3 (mod 4)$ nên $-4$ là số chính phương mod $p$ hay $-1$ là số chính phương mod $p$ suy ra $p \equiv 1 (mod 4)$ mâu thuẫn với $p \equiv 3 (mod 4)$

Vd3: Chứng minh rằng phương trình $y^2=x^3-6$ không có nghiệm tự nhiên.
Giải:

Giả sử ngược lại. Nếu $x$ chẵn thì $y^2 \equiv -6 \equiv 2 (mod 8)$ ( vô lí với số chính phương )
Nên $x$ lẻ, $y$ lẻ và $x^3=y^2+6 \equiv 7 (mod 8)$ Ta cũng có: $x^3 \equiv x (mod 8 ) \forall x$ lẻ, nên $x \equiv 7 (mod 8)$
Viết lại:
$y^2-2=(x-2)(x^2+2x+4)$ với $x^2+2x+4 \equiv 7^2+2.7+4 \equiv 3 (mod 8)$. Vì thế phải có ước $p \equiv \pm 3 (mod 8)$ vì nếu không $x^2+2x+4 \equiv \pm 1 (mod 8)$. Ta có $2 \equiv y^2 (mod p) \Rightarrow p \equiv \pm 1 (mod 8)$ ( Mâu thuẫn)


Thứ Năm, 23 tháng 3, 2017

Dùng định lý hàm số sin để chứng minh song song

Cho tam giác $ABC$ nội tiếp đường tròn (O) và ngoại tiếp đường tròn $(I)$. $(K)$ là đường tròn Mixtilinear góc A của tam giác $ABC$ tiếp xúc $(O)$, $AC,AB$ lần lượt tại $X, A_c, A_b$, $AX\cap A_bA_c= Y, XI \cap BC =K$. CMR: $KY \parallel AI$

Giải

Theo định lý Lyness $A_b$, $A_c$ và $I$ thẳng hàng. Vì $I$ là trung điểm $A_cA_b$ và $AX$ là đường đối trung của góc $\angle A_bXA_c$ ta có: $\measuredangle A_cXI= \measuredangle AXA_b = \measuredangle A_cCI$, nên $A_cCXI$ nội tiếp. tương tự $A_bBXI$ nội tiếp. $$\frac{XK}{KI} = \frac{XC}{CI} \cdot \frac{ \sin \angle BCX}{ \sin \angle ICB} = \frac{ \sin \angle XIC}{ \sin \angle IXC}\cdot \frac{ \sin \angle BCX}{ \sin \angle ICB}$$ $$\frac{ \sin \angle XIC}{ \sin \angle IXC}\cdot \frac{ \sin \angle BCX}{ \sin \angle ICB} = \frac{ \sin \angle XA_bY}{ \sin \angle AA_bY}\cdot \frac{ \sin \angle BAX}{ \sin \angle AXA_b}$$ $$\frac{ \sin \angle XA_bY}{ \sin \angle AA_bY}\cdot \frac{ \sin \angle BAX}{ \sin \angle AXA_b} = \frac{XY}{YA}$$ $\Longrightarrow \frac{XK}{KI} = \frac{XY}{YA} \Longrightarrow AI \parallel YK$

Thứ Hai, 13 tháng 3, 2017

Dùng điều kiện toàn ánh để giải phương trình hàm

Bài 1(Iran TST 2011): Tìm tất cả hàm số toàn ánh: $f:\mathbb{R} \mapsto \mathbb{R}$ thỏa mãn:

\[f(x+f(x)+2f(y))=f(2x)+f(2y).(\forall x,y \in \mathbb{R}) \]

Lời giải:

Do f toàn ánh nên tồn tại a sao cho f(a)=0.

$(a,a)$ $\Rightarrow $ $f(2a)=0$ $\Rightarrow$ $f(4a)=0$ $\Rightarrow $ ....

Từ (a,y) và (2a,y) và tính toàn ánh của hàm số f ta suy ra:

$f(x+a)=f(x) \forall x \in R \Rightarrow  f(x)=f(x-a) \forall x \in R$ (1)

$\Rightarrow f(0)=f(a)=0$

Do f là toàn ánh nên với mỗi x thuộc R tồn tại $y_o$ sao cho: $f(y_o)=\frac{x-f(x)}{2}$
$(x,y_o) \Rightarrow f(2y_o)=0 $

$(0,y_o) \Rightarrow f(2y_o)=0=f(2f(y_o))=f(x-f(x))=0 (\forall x \in R)$
Tương tự như (1) ta có:
$f(x)=f(x-(x-f(x)))=f(f(x)) (\forall x \in R) $ Do f toàn ánh nên $f(x)=x (\forall x \in R)$
Nhận xét: Ý tưởng: "tồn tại $y_o$ sao cho: $f(y_o)=\frac{x-f(x)}{2}$" khá quan trọng trong bài toán
Bài 2: (30/04 lớp 11 năm 2016):  Tìm tất cả hàm số toàn ánh: $f:\mathbb{R} \mapsto \mathbb{R}$ thỏa mãn:
\[f(x+f(x)+2f(y)+2f(z))=f(2x)+f(2y)+f(2z).(\forall x,y,z \in \mathbb{R}) \] 

Lời giải:
Do f toàn ánh nên tồn tại a sao cho f(a)=0.
Cho $x=y=z=a$ ta được $f(2a)=0$
Cho $z=a$ ta được đề Iran TST 2011.

Bài 3: (Brazil 2006): Tìm tất cả hàm số: $f\colon \mathbb{R}\to \mathbb{R}$ thỏa mãn:
\[f(xf(y)+f(x)) = 2f(x)+xy\] với mọi số thực x,y.

Lời giải:

Cho $x=1$ Dễ dàng suy ra được hàm số đã cho song ánh.
Nên tồn tại a,b sao cho $f(a)=0, f(b)=1$
Cho $x=a, y=b$ ta được:
$ab=0$
Nếu $a=0$ thì cho y=0 vào ta suy ra $f(f(x))=2f(x)$ với mọi x thực dùng điều kiện song ánh suy ra $f(x)=2x \forall x \in R$ thử lại thấy không thỏa
Vậy $b=0$.
Mặt khác cho $x=y=-1$ vào pt hàm ban đầu ta được:
$f(-1)=0$

Cho y=-1 vào pt hàm ban đầu ta được:
$f(f(x))=2f(x)-x$ (2)
Trong (2) cho x=0 ta được $f(1)=2$
Cho x=-1,y=1 ta được:
$f(-2)=-1$ 

Mặt khác do f là toàn ánh nên với mỗi x thuộc R tồn tại $y_o$ sao cho $f(y_o)=f(x)-x $
$P(x,-2): f(f(x)-x)=2(f(x)-x) $hay $f(f(y_o))=2f(y_o)$
Mặt khác từ (2) suy ra $f(f(y_o))=2f(y_o)-y_o$
Suy ra $y_o=0$ suy ra $f(x)=x+1$ với mọi x thuộc R

Nhận xét: ta thấy được $f(x)=x+1$ thỏa mãn yêu cầu đề bài và ta cần tìm y sao cho $2(xf(y)+f(x))=2f(x)+xy$ hay $2f(y)=y$ giải phương trình được $y=-2$

Bài tập: các bạn hãy dùng cách trên để giải VMO 2017:
Tìm tất cả các hàm số : f:RR thỏa mãn hệ thức:
$$f\left ( xf\left ( y \right )-f\left ( x \right ) \right )=2f\left ( x \right )+xy$$ với mọi số thực 

Thứ Hai, 6 tháng 2, 2017

Tính chất của dãy số Fibonacci

1) $(F_n,F_{n+1})=1$

2) Nếu $n |m $ thì $F_n |F_m$
Ta chỉ cần chứng minh tính chất sau:
$F_{m+n}=F_{m-1}F_{n+1}+F_{m}.F_{n}$
Quy nạp theo $n$, với $n=1$ đúng
Giả sử đúng với $n=k$ khi đó với $n=k+1$ thì:
$F_{m+k+1}=F_{m+k}+F_{m+k-1}=(F_{m-1}F_{k+1}+F_{m}.F_{k})+(F_{m-1}_F{k}+F_{m}.F_{k-1})=F_{m-1}F_{k+2}+F_{m}F_{k+1}$
Vậy theo nguyên lí quy nạp ta có điều phải chứng minh.
Cho $m=kn$ thì ta suy ra thêm được một số tính chất sau
3)Nếu $F_n$ chia hết cho $F_m$ thì $n$ chia hết cho $m$ (m>2)
4) $(F_m,F_n)=F_{(m,n)}$
5) $n \ge 5$ và $F_n$ là số nguyên tố thì n cũng là số nguyên tố.
6) $(F_n)$ chứa vô hạn những số nguyên tố đôi một cùng nhau
7) $F_{5n}=5F_nq_n$ $q_n$ không chia hết cho 5.

Chứng minh:

Cách 1:
$F_{5n}=\frac{q_1^{5n}-q_2^{5n}}{\sqrt 5}=F_n(q_1^{4n}+q_1^{3n}q_2^n+(q_1q_2)^2n+q_1^nq_2^{3n}+q_2^{4n})=F_n(L_{4n}+(-1)^nL_{2n}+1)=F_n(L_{2n}^2+(-1)^nL_{2n}-1).$ Vì thế $v_5(F_{5n})=v_5(F_n)+v_5(L_{2n}^2+(-1)^nL_{2n}-1)$.
$L_n^2-5F_n^2=4(-1)^n$
$F_{2n}=F_nL_n$.
Do $n=1$ $L_2^2-L_2-1=5$.
$L_n,F_n$ chu kì 20 mod 5 ($L{n+10}=-L_n\mod 5, F_{n+10}=-F_n\mod 5$. $5|F_n$ khi và chỉ khi $5|n$
nếu $n>1$ $5|F_5|F_{5k}$. Vì thế $5|n$  $L_{2n}^2=4(-1)^n\mod 25$. Vì $10|n$ $L_{2n}=\pm 2\mod 25$.
$L_{2n}^2+(-1)^nL_{2n}-1=4+2-1=5\mod 25$. Vậy $v_5(F_{5n})=v_5(F_n)+1\to v_5(F_n)=v_5(n).$ (Đpcm)

Cách 2:



Dùng cách tính chất ở trên, nếu $a|b$ thì $F_a|F_b$ và , $(F_a,F_b)=F_{(a,b)}$.

Đặt $n=5^p \cdot q$ Với $(5,q)=1$. thì $v_5(n)=p$. thấy rằng $(F_{5^k \cdot m}, F_{5^k}) = F_{5^k}$ . Hiển nhiên $5^k|F_{5^k}$.và $5^{k+1}$ không là ước của ${F_{5^k \cdot m}}$ vì nó sẽ dẫn đến $5|m$ mâu thuẫn. Vậy $v_5(F_{5^k \cdot m})=k$ . Đpcm $\Box$




8) $F_n \vdots 5^k$ khi và chỉ khi $n \vdots k$
9) $F_n$ có tận cùng là 0 khi và chỉ khi $n \vdots 15$

10) $F_n$ có tận cùng là hai chữ số 0 khi và chỉ khi $n \vdots 150$


Bất đẳng thức tuyển sinh lớp 10 chọn lọc

Trong bài viết này, tác giả giới thiệu một số bài BĐT nhẹ nhàng nhưng ý tưởng tương đối mới, mức độ phù hợp với đề thi tuyển sinh vào lớp...